If I am given a region $R$ defined as the the region between a unit disk $D_1(0)$ and $D_{ \frac{1}{4}}$$(\frac{1}{4})$, what is a strategy to conformally map this to an annulus $a \lt z \lt 1$? Specifically, where does the constraint on $a$ come from?
Finding a conformal map between an irregular region between two disks and an annulus
1
$\begingroup$
complex-analysis
-
1This is very similar to http://math.stackexchange.com/questions/238205/complex-analysis-simultaneous-mapping – 2012-11-16